How to represent a periodic function using Taylor series

Click For Summary
Periodic functions like sine and cosine can be represented using Taylor series, but not all periodic functions can be. Specifically, a triangular wave is not differentiable, which makes it unsuitable for representation by a Taylor series. While trigonometric series are appropriate for periodic functions, the Taylor series is limited to functions that are smooth and differentiable. Therefore, using a Taylor series for a triangular wave or square wave is not feasible. Understanding the properties of the function is crucial for determining the appropriate series representation.
PainterGuy
Messages
938
Reaction score
73
Hi,

Is this possible to represent a periodic function like a triangular wave or square wave using a Taylor series? A triangular wave could be represented as f(x)=|x|=x 0<x<π or f(x)=|x|=-x -π<x<0. I don't see any way of doing although I know that trigonometric series could be used instead.

This is not a homework question so I don't think I was supposed to use the template. If my question doesn't fit in this section, please move it to relevant section. Thank you for your help.
 
Physics news on Phys.org
PainterGuy said:
Hi,

Is this possible to represent a periodic function like a triangular wave or square wave using a Taylor series? A triangular wave could be represented as f(x)=|x|=x 0<x<π or f(x)=|x|=-x -π<x<0. I don't see any way of doing although I know that trigonometric series could be used instead.

This is not a homework question so I don't think I was supposed to use the template. If my question doesn't fit in this section, please move it to relevant section. Thank you for your help.

You can represent periodic function like sine and cosine with Taylor series - so being periodic doesn't make it impossible. On the other hand a triangular wave isn't even differentiable - so no on that one.
 
  • Like
Likes PainterGuy
Question: A clock's minute hand has length 4 and its hour hand has length 3. What is the distance between the tips at the moment when it is increasing most rapidly?(Putnam Exam Question) Answer: Making assumption that both the hands moves at constant angular velocities, the answer is ## \sqrt{7} .## But don't you think this assumption is somewhat doubtful and wrong?

Similar threads

  • · Replies 6 ·
Replies
6
Views
2K
Replies
4
Views
2K
  • · Replies 27 ·
Replies
27
Views
3K
  • · Replies 2 ·
Replies
2
Views
2K
  • · Replies 3 ·
Replies
3
Views
2K
Replies
6
Views
3K
  • · Replies 10 ·
Replies
10
Views
2K
Replies
5
Views
2K
  • · Replies 11 ·
Replies
11
Views
3K
Replies
4
Views
2K